Đến nội dung

Cantho2015 nội dung

Có 47 mục bởi Cantho2015 (Tìm giới hạn từ 29-04-2020)



Sắp theo                Sắp xếp  

#563465 Tìm quỹ tích trực tâm H của tam giác ABC

Đã gửi bởi Cantho2015 on 04-06-2015 - 16:27 trong Hình học

Cho điểm P cố định nằm ngoài đường tròn (O). Kẻ tiếp tuyến PA và cát tuyến bất kì PBC. Tìm quỹ tích trực tâm H của tam giác ABC




#641183 Tìm nghiệm nguyên dương của phương trình: $$x^3-y^3=xy+61$...

Đã gửi bởi Cantho2015 on 19-06-2016 - 03:48 trong Đại số

Bài A xem ở đây http://diendantoanho...ên-a3-b3-ab-61/




#645566 Tìm giá trị nhỏ nhất của $T=a+b+c+\frac{1}{abc}...

Đã gửi bởi Cantho2015 on 19-07-2016 - 18:14 trong Bất đẳng thức và cực trị

$P=(a+b+c+\frac{1}{9abc})+\frac{8}{9abc}\geq 4.\sqrt[4]{abc.\frac{1}{9abc}}+\frac{8}{9(\sqrt{\frac{a^{2}+b^{2}+c^{2}}{3}})^{3}}= 4\sqrt{3}$

Dấu = xảy ra <=> a = b = c = $\frac{1}{\sqrt{3}}$

 

$abc \leq (\sqrt{\frac{a^2+b^2+c^2}{3}})^3$ ngoài cách chứng minh bằng bđt Holder $3(a^2+b^2+c^2) \geq (a+b+c)^2$ còn cách nào đơn giản hơn không bạn?




#645545 Tìm giá trị nhỏ nhất của $T=a+b+c+\frac{1}{abc}...

Đã gửi bởi Cantho2015 on 19-07-2016 - 16:59 trong Bất đẳng thức và cực trị

Cho $a,b,c>0$ và $a^2+b^2+c^2=1$

Tìm giá trị nhỏ nhất của $T=a+b+c+\frac{1}{abc}$




#642107 Topic: Thảo luận về các bài tập trong chuyên đề số học của VMF.

Đã gửi bởi Cantho2015 on 25-06-2016 - 11:47 trong Số học

Việc làm rất ý nghĩa. T ủng hộ một bài:

 

b) Đặt $(m,n)=k$ ta có $m=m'k, n=n'k$ với $(m',n')=1$.

Khi đó $2^m-1=2^{m'k}-1=(2^k-1)P$. Tương tự $2^n-1=2^{n'k}-1=(2^k-1)Q$.

Chú ý $(m',n')=1$ nên $(P,Q)=1$ tức $(2^m-1; 2^n+1)=2^k-1=2^{(m,n)-1}$.

Cái chỗ $(m',n')=1 \Rightarrow (P,Q)=1$ em không hiểu lắm. Với em nghĩ $(2^m-1;2^n-1)=2^k-1$ chứ đâu phải $(2^m-1;2^n+1)=2^k-1$ 




#642104 Topic: Thảo luận về các bài tập trong chuyên đề số học của VMF.

Đã gửi bởi Cantho2015 on 25-06-2016 - 11:28 trong Số học

Bài 13:

Dãy Fermat phải là $$F_n=2^{2^n}+1$$ và có thể tổng quát lên là dãy $F_k=a^{2^k}+b^{2^k}$ với $gcd(a;b)=1$, $2|ab$ và $k \in \mathbb{Z^+}$

Giải: Vì $2|ab$ và $(a;b)=1$ nên giả sử $a$ là số chẵn thì $b$ phải là số lẻ. Giả sử $m>n$ với $m,n$ là các số tự nhiên bất kỳ. 
Gọi $(F_m;F_n)=d$ (d là số lẻ vì cả hai $F_m,F_n$ lẻ)
Ta có $a^{2^{n+1}}-b^{2^{n+1}}|a^{2^{n+1}2^{m-n-1}}-b^{2^{n+1}2^{m-n-1}}$ vì $(x-y|x^k-y^k)$ ở đây $(x=a^{2^{n+1}}, y=b^{2^{n+1}}, k=m-n-1)$
Mà $a^{2^{n}}+b^{2^{n}}|(a^{2^{n}})^2-(b^{2^{n}})^2$ suy ra $F_n|F_m-2b^{2^{m}}$
Từ $d|F_n$ và $d|F_m$ suy ra $d|2b^{2^{m}} \Rightarrow d|b^{2^{m}}$. Mà $d|F_m$ suy ra $d|a^{2^{m}}$ kết hợp với giả thuyết $(a;b)=1 \Rightarrow d=1$
(Với dãy Fermat $d|2 \Rightarrow d=1$ nhanh hơn tí)
 



#643435 Topic tổng hợp các bài toán về phương trình nghiệm nguyên.

Đã gửi bởi Cantho2015 on 03-07-2016 - 13:27 trong Số học

Tìm nghiệm nguyên dương của phương trình: $a^2+b^2+c^2=2(ab+bc+ca)$

$a^2+b^2+c^2=2(ab+bc+ca) \Leftrightarrow (a-b)^2+(b-c)^2+(c-a)^2=a^2+b^2+c^2$

$\Rightarrow$ $\begin{cases} a-b=a\\c-a=a \end{cases}$ $\Rightarrow \begin{cases} a=0, b=c\\ a=0, b=c \end{cases}$

Hoặc $b-c=a \Rightarrow$ $\begin{cases} b-c=b\\c-a=b \end{cases}$ $\Rightarrow \begin{cases} c=0,b=a\\a=0, b=c \end{cases}$

Xét tương tự ta suy ra nghiệm pt là $a=0, b=c$ và $b=0, a=c$ và $c=0, b=a$




#640977 Topic tổng hợp các bài toán về phương trình nghiệm nguyên.

Đã gửi bởi Cantho2015 on 18-06-2016 - 02:39 trong Số học

Tiếp nè:

     Tìm tất cả các cặp số nguyên không âm thoả mãn phương trình: $(y+1)^{4}+y^{4}=(x+1)^{2}+x^{2}$

 
 $$(y+1)^4+y^4=(x+1)^2+x^2$$
Giải:
 $\Leftrightarrow 2x^2+2x+1-(y+1)^4-y^4=0$
 $\Rightarrow \Delta_x=4-8(2y^4+4y^3+6y^2+4y)=k^2$ 
Vì vế trái luôn chia hết cho 2
 $\Rightarrow k=2m \Rightarrow 1-2(2y^4+4y^3+6y^2+4y)=m^2$
     Kẹp!
  Ta có:
 $ [1-2(2y^4+4y^3+6y^2+4y)]-(2y^2+2y+1)^2=4y^2+4y=4y(y+1) \geq{0}$, $\forall y \in \mathbb{Z}$ 
  Lại có:
 $ (2y^2+2y+2)^2 -[1-2(2y^4+4y^3+6y^2+4y)]=3 > 0$
 $ \Rightarrow(2y^2+2y+1)^2 \leq m^2 < (2y^2+2y+2)^2$
 $ \Rightarrow (2y^2+2y+1)^2=m^2 \Leftrightarrow 4y(y+1)=0 \Leftrightarrow y=0, y=-1$
 $ \Rightarrow (x;y)=(0;0),(-1;0),(-1;0),(-1;-1)$



#641347 Topic tổng hợp các bài toán về phương trình nghiệm nguyên.

Đã gửi bởi Cantho2015 on 20-06-2016 - 04:57 trong Số học

mọi người xem giúp mình bài này với ..giair phương trình với x,y nguyên: $x^y=y^x$

Mình nghĩ là x,y phải nguyên dương

Nếu x,y nguyên dương, không mất tính tổng quát, giả sử $x>y$ và $(x;y)=d (d>0)$

$\Rightarrow x=dy$

pt $\Leftrightarrow (dy)^y-(y^d)^y=[dy-(y^d)]A (A>0)$

$\Rightarrow dy=y^d$

Với $d=1$ suy ra $x=y$

Với $d>1$, dễ thấy $y^d>dy$, vậy pt vô nghiệm




#640833 Topic tổng hợp các bài toán về phương trình nghiệm nguyên.

Đã gửi bởi Cantho2015 on 17-06-2016 - 11:46 trong Số học

Tìm số nguyên x để $2{{\text{x}}^{2}}-x-36$ là bình phương một số nguyên tố

$2x^2-x-36=(2x-9)(x+4)=p^2$ (p là số nguyên tố)

$\Rightarrow 2x-9=x+4$ hoặc $2x-9= \pm{1}$ hoặc $x+4= \pm{1}$

Giải ra được $(x;p)=(13;17),(5;3)$




#640978 Topic tổng hợp các bài toán về phương trình nghiệm nguyên.

Đã gửi bởi Cantho2015 on 18-06-2016 - 04:14 trong Số học

Tìm $x , y \geq 0$ biết $\left ( xy - 7 \right )^{2} = x^{2} + y^{2}$

 

Giải:

$(xy-7)^2=x^2+y^2 \Leftrightarrow (xy-7)^2+2(xy-7)+1=x^2+y^2+2xy-13$

$(xy-6)^2-(x+y)^2=-13=13.(-1)$

Vì $x,y \geq 0$ nên $xy-6+(x+y) \geq xy-6-(x+y)$

$ \Rightarrow$

$$\begin{cases} xy-6+(x+y)=13\\xy-6-(x+y)=-1 \end{cases}$$

$ \Rightarrow$

$$\begin{cases} xy=6\\x+y=7 \end{cases}$$

$\Rightarrow$ $(x;y)=(3;4),(4;3)$




#643358 Topic tổng hợp các bài toán về phương trình nghiệm nguyên.

Đã gửi bởi Cantho2015 on 02-07-2016 - 22:00 trong Số học

Mình nghĩ là x,y phải nguyên dương

Nếu x,y nguyên dương, không mất tính tổng quát, giả sử $x>y$ và $(x;y)=d (d>0)$

$\Rightarrow x=dy$

pt $\Leftrightarrow (dy)^y-(y^d)^y=[dy-(y^d)]A (A>0)$

$\Rightarrow dy=y^d$

Với $d=1$ suy ra $x=y$

Với $d>1$, dễ thấy $y^d>dy$, vậy pt vô nghiệm

Bài giải thiếu nghiệm với phần chứng minh $x=dy$ mình bổ sung:

$gcd(x,y)=1 \Rightarrow x=dx_1, y=dy_1$ với $ gcd(x_1,y_1)=1$ và $x_1 > y_1$ ( vì $x>y$, $x=y$ thì xét riêng)

$x^y=y^x \Leftrightarrow [(dx_1)^{y_1})^d-[(dy_1)^{x_1}]^d=0 \Leftrightarrow ((dx_1)^{y_1}-(dy_1)^{x_1})B=0$

Vì $B>0$ nên $(dx_1)^{y_1}=(dy_1)^{x_1} \Rightarrow (x_1)^{y_1}=d^{x_1-y_1}(y_1)^{x_1} \Rightarrow y_1|x_1 \Rightarrow y_1=1 \Rightarrow y=d \Rightarrow x=ky$ 

Sau đó giải như trên, suy ra $ky=y^k$. 

Với $k=y=2 \Rightarrow x=4$

Với $k>2$ thì vế phải tăng nhanh hơn vế trái nên vô nghiệm 

Vậy nghiệm là $x=y$ hoặc $(x;y)=(2;4),(4;2)$




#641184 Giải pt nghiệm nguyên: $x^2+y^2+xy=x^2y^2$

Đã gửi bởi Cantho2015 on 19-06-2016 - 04:36 trong Số học

Phương trình tương đương $(x+y)^2=xy(xy+1)$
Từ đây suy ra $(x;y)=(0;0),(1;-1),(-1;1)$.

Tại sao $(x+y)^2=xy(xy+1)$ thì suy ra được nghiệm vậy bạn?




#643437 giải pt nghiệm nguyên $x^2+y^2+z^2=xyz$

Đã gửi bởi Cantho2015 on 03-07-2016 - 13:44 trong Số học

áp dụng BĐT côsi cho 3 số

$x^{2}+y^{2}+z^{2}\geqslant 3\sqrt[3]{x^{2}y^{2}z^{2}}$

dấu = khi và chỉ khi x=y=z

x2 + y2 +z2 = xyz

$xyz\geqslant 3\sqrt[3]{x^{2}y^{2}z^{2}}$

$\Leftrightarrow$ $\frac{x^{3}y^{3}z^{3}}{27}\geqslant (xyz)^{2}$

dấu = khi x=y=z suy ra:

$\frac{x^{3}y^{3}z^{3}}{27}- (xyz)^{2}=0$

giải pt ta đc:

x = y = z =0             ( chọn)

hoặc x = y =z = 3    ( chọn)

Nếu dấu "=" không xảy ra thì sao bạn?




#641185 Giải phương trình nghiệm nguyên $x^2-3y^2+2xy-2x+6y-8=0$

Đã gửi bởi Cantho2015 on 19-06-2016 - 05:12 trong Số học

Đưa về phương trình ước số! Anh ơi cho em hỏi, mình có kinh nghiệm gì để biết phương trình nào đưa về pt ước số hay không và có cách nào dễ dàng đưa về phương trình ước số!!!?

pt $\Leftrightarrow x^2+2x(y-1)-3y^2+6y-8=0$

$\Delta_{x}=4(y-1)^2-4(-3y^2+6y-8)=(4y)^2-32y+35$

Ta muốn $\Delta_{x}$ là một số chính phương, để ý thấy $-32y=-2.4.4y$ mà $(a-b)^2=a^2-2ab+b^2$ ta đã có $a^2=(4y)^2$ nên $b=4$ hay $b^2=16$.

Trong những phương trình nghiệm nguyên kiểu này, ta có một lợi thế là ta có thể điều chỉnh hệ số tự do tùy thích. Vậy đừng lấy $-8$ nữa. Lấy $a$ sao cho $4-4a=16$ ($4$ có được từ hệ số tự do của $4(y-1)^2$). Thấy ngay là $a=-3$.

Vậy ta có $\Delta_x=(4y-4)^2$, vậy $x=\frac{-(2y-2) \pm (4y-4)}{2}$

$\Rightarrow x=3y-3$ hoặc $x=-y+1$

Mà theo định lý Bezout thì nếu x có nghiệm như trên ta phân tích được $[x-(-y+1)][x-(3y-3)]-5=0$ ($-5=-8+3$)

Vậy "trick" của bài này là chọn hệ số sao cho $\Delta_{x}$ là bình phương của một số thôi và ta cho hệ số $-3$




#641211 Giải phương trình nghiệm nguyên $(x^2+y)(y^2+x) = (x-y)^3$

Đã gửi bởi Cantho2015 on 19-06-2016 - 09:31 trong Số học

Xem bài giải ở đây http://diendantoanho...cho-x2yy2xx-y3/

 




#643613 CMR: $4^{x} + x + y \vdots 6$

Đã gửi bởi Cantho2015 on 04-07-2016 - 12:32 trong Số học

Ta có $A=4^x+x+y=4^x-2014+(x+1)+(y+2013)$

$A=(2^x)^2-2^2-(6)(335)+(x+1)+(y+2013)=(2^x-2)(2^x+2)+6k$

Vì $2^x-2$ và $2^x+2$ là bội của $2$ nên tích của chúng chia hết cho $2$.

Ta có $2 \equiv -1 \pmod{3}$

Nếu $x$ lẻ suy ra $2^x-2$ chia hết cho $3$, $x$ chẵn suy ra $2^x+2$ chia hết cho $3$.

Vì $(2^x-2)(x^x+2)$ chia hết cho $3$ và $2$ nên chia hết cho $6$ $\Rightarrow$ đpcm




#640984 Chứng minh rằng phương trình: $x^2+5=y^3$ không có nghiệm nguyên

Đã gửi bởi Cantho2015 on 18-06-2016 - 07:37 trong Số học

Chứng minh rằng phương trình: $x^2+5=y^3$ không có nghiệm nguyên

Bổ đề:

Nếu $4k+3|x^2+y^2$ thì $4k+3|x$ và $4k+3|x$ (chứng minh bằng phản chứng và định lý Fermat nhỏ)

 

Vì $x^2 \equiv{0;1} \pmod{4} \Rightarrow y^3=x^2+5 \equiv{1;2} \pmod{4}$. Vậy $y^3$ không chia hết cho 4, vậy $y$ không chia hết cho 2 hay y là số lẻ. 

Nếu $y \equiv{3} \pmod{4}$ thì $y^3 \equiv{3} \pmod{4}$ hay $x^2 \equiv{2} \pmod{4}$ $\Rightarrow$ vô lý

Nếu $y \equiv{2} \pmod{4}$ thì $y^3 \equiv{0} \pmod{4}$ $\Rightarrow$ vô lý

$\Rightarrow y=4k+1$

$(x^2+(2)^2=y^3-1=(y-1)(y^2+y+1)$

$ \Rightarrow y^2+y+1=4(4k^2+3k)+3 \equiv{3} \pmod{4}$ 

$ \Rightarrow 4m+3|x^2+(2)^2 \Rightarrow$ vô lý




#550493 Chứng minh rằng MO là tia phân giác của góc KMF

Đã gửi bởi Cantho2015 on 31-03-2015 - 16:56 trong Hình học

Ta có: $\Delta IKA \sim \Delta IAB => \frac{IK}{IA}=\frac{IA}{IB}=> \frac{IK}{IM}=\frac{IM}{IB}$
=> $\Delta IMK \sim \Delta IBM => \angle IMK = \angle IBM$
Ta có: $\Delta MKA \sim \Delta AKB => \frac{AK}{MK}=\frac{AB}{AM}=\frac{2AH}{2IM}=\frac{AH}{IM}$ 
=> $\Delta IMK \sim \Delta HAK =>  \angle KHA= \angle MIK$ => tứ giác IKHA nội tiếp => $\angle MAB = \angle HKB$
=> cung AB = cung BF => AB = BF => $\frac{AK}{AM}=\frac{KB}{AB}=\frac{AK}{BM}=\frac{KB}{BF}$
Kết hợp với $\angle MBF = \angle AKB$ => $\Delta AKB \sim \Delta MBF$ => $\angle KAB = \angle BMF$ 
Mà $\angle KAB = \angle IBM$ => $\angle IMK = \angle BMF$
Mà MO phân giác  $\angle AMB$ => $\angle KMH = \angle FMH$ => MH phân giác $\angle KMF$

Đúng rồi mà hơi dài tí. Gọi giao điểm của MF với (O) là G. Nếu ta chứng minh được AG là đường trung tuyến thứ 2 của tam giác MAB thì ta sẽ dễ dàng chứng minh được tam giác MKG cân tại M suy ra đpcm. Bạn giúp mình cái



#550252 Chứng minh rằng MO là tia phân giác của góc KMF

Đã gửi bởi Cantho2015 on 30-03-2015 - 16:10 trong Hình học

Cho đường tròn (O;R) vẽ dây cung AB< 2R. Các tiếp tuyến Ax,By của (O;R) cắt nhau tại M. Gọi I là trung điểm của MA và K là giao điểm của BI với (O). Gọi H là giao điểm của MO và AB. Kẻ dây cung KF đi qua điểm H. Chứng minh rằng MO là tia phân giác của góc KMF



#639965 Chứng minh rằng $11^{10^{1967}}-1$ chia hết cho...

Đã gửi bởi Cantho2015 on 13-06-2016 - 03:42 trong Số học

Ta có

$11^{{10}^{1967}}-1^{{10}^{1967}}=(11-1)(11^{{10}^{1967}-1}+11^{{10}^{1967}-2}+...+1)$

$11\equiv 1\pmod{10}\\$$\Rightarrow 11^k\equiv 1\pmod {10}$, với k $>$ 0

$\Rightarrow(10)(11^{{10}^{1967}}+...+1)\equiv (10)(1+1+...+1)\equiv 10(10^{1967})\equiv 0\pmod{10^{1968}}$




#563878 Chứng minh nếu MN vuông góc DP thì tam giác AND cân

Đã gửi bởi Cantho2015 on 06-06-2015 - 09:00 trong Hình học

1/Cho hình chữ nhật ABCD có M là trung điểm AB, N thuộc tia phân giác góc BCD. Gọi P là hình chiếu của N trên BC. Chứng minh nếu MN vuông góc DP thì tam giác AND cân.

 

2/Cho đường trong (O) nội tiếp tam giác ABC và tiếp xúc BC,CA,AB lần lượt tại D,E,F. Chứng minh rằng tích các khoảng cách hạ từ một điểm M bất kỳ trên đường tròn xuống các cạnh tam giác ABC bằng tích khoảng cách từ M đến các cạnh tam giác DEF.

 

3/ Cho hình vuông ABCD. Điểm M thuộc cạnh AB ( M khác A và B). Tia CM cắt DA tại N. Vẽ tia Cx vuông góc với CM và cắt tia AB tại E. Gọi H là trung điểm NE. 1) chứng minh tứ giác BCEH nội tiếp. 2) Tìm vị trí M để diện tích tứ giác NACE gấp ba lần diện tích hình vuông ABCD. 3) Chứng minh khi M di chuyển trên AB thì tỉ số bán kính các đường tròn nội tiếp tam gái NAC và tam giác HBC không đổi




#641225 Chứng minh $x(x+1)=p^{2n}(y+1)$ vô nghiệm

Đã gửi bởi Cantho2015 on 19-06-2016 - 10:35 trong Số học

1. Chứng minh phương trình nghiệm nguyên $x(x+1)=p^{2n}y(y+1)$ vô nghiệm với $p$ là số nguyên tố và $n$ là số nguyên dương

2. Giải phương trình nghiệm nguyên với p là số nguyên tố, $x,y$ là hai số nguyên dương

$x^5+x^4+1=p^y$

3. Giải hệ phương trình nghiệm nguyên

$\begin{cases}x+y+z+u+v=xyuv+(x+y)(u+v)\\xy+z+uv=xy(u+v)+uv(x+y)\end{cases}$

 




#641344 Chứng minh $x(x+1)=p^{2n}(y+1)$ vô nghiệm

Đã gửi bởi Cantho2015 on 20-06-2016 - 01:49 trong Số học

Bài 1 mình giải thế này:

  $$\displaystyle{\frac{x(x+1)}{y(y+1)}=(p^n)^2}$$
 Dễ thấy \[gcd(x;x+1)=gcd(y;y+1)=1 \]
Giả sử $y|x \Rightarrow y \nmid x+1, y+1 \nmid x$
  $$x=my; x+1=n(y+1) $$ (với $m,n \geq{1}$)
  $(m;n)=d \Rightarrow d|my-n(y+1) \Rightarrow d|x-(x+1) \Rightarrow d=1$
  $mn=(p^n)^2=p.p.p....p$ $\Rightarrow$ Một trong $m$ hoặc $n$ phải bằng $1$ 
 Cả hai trường hợp $\Rightarrow x=y$. Vậy $p=1$ (vô lý)
  Nếu $y|x+1$, chứng minh tương tự



#641346 Chứng minh $x(x+1)=p^{2n}(y+1)$ vô nghiệm

Đã gửi bởi Cantho2015 on 20-06-2016 - 02:27 trong Số học

Bài 2 quan trọng là phát hiện:

$x^5+x^4+1=x^5+x^3+x^3+1-x^3=(x^2+x+1)(x^3-x+1)$

Viết pt lại: $(x^2+x+1)(x^3-x+1)=p^y$

Dễ thấy $x$ phải dương

Nếu $x=1$ thì $p=3,y=1$

Nếu $x>1$ thì khi đó $y>1$ (Do $x^2+x+1>1, x^3-x+1$)

Giả sử $(x^2+x+1,x^3-x+1)=d$

Suy ra $d | -(x^2+x+1+x^3-x+1)+x(x^2+x+1)=x-2$

$d| -(x-2)(x+3)+(x^2+x+1)=5$

Nên nếu $p=5$, thì $(x^2+x+1,x^3-x+1)=5$

 Mà $x^2+x+1 \ne 5$ và $x^3-x+1 \ne 5$ (với $x$ nguyên)

Nên $p=5$ phương trình vô nghiệm.

$p \ne 5$ thì suy ra: $(x^2+x+1,x^3-x+1)=1$ Mà $x^2+x+1 \ne 1$ và $x^3-x+1 \ne 1$ (với $x>1$ )

Vậy: $(x,y,p)=(1,1,3)$

Cái chỗ $(x^2+x+1,x^3-x+1)=5$ $\Rightarrow$ $x^2+x+1 \ne 5$ sao không phải là $x^2+x+1 \ne 5k$, mình không hiểu.